Đến nội dung

nhungvienkimcuong nội dung

Có 206 mục bởi nhungvienkimcuong (Tìm giới hạn từ 15-05-2020)



Sắp theo                Sắp xếp  

#731842 Đề thi chọn đội tuyển lớp 12 - VMO - Bà Rịa-Vũng Tàu 2022

Đã gửi bởi nhungvienkimcuong on 30-11-2021 - 20:19 trong Thi HSG cấp Tỉnh, Thành phố. Olympic 30-4. Đề thi và kiểm tra đội tuyển các cấp.

Bài 1 (4,0 điểm).

b) Tìm tất cả bộ ba số thực dương $(x,y,z)$ thoả mãn hai điều kiện $xy+yz+zx+xyz=4$ và $\sqrt{2(4-xy)}+ \sqrt{5(4-yz)}+ \sqrt{10(4-zx)}=12$.

 

Với điều kiện $xy+yz+zx+xyz=4$ thì tồn tại các số dương $a,b,c$ sao cho (xem thêm ở đây)

$$x=\frac{2b}{a+c},\ y=\frac{2c}{a+b},\ z=\frac{2a}{b+c}$$

Thay vào điện kiện thứ hai ta có được

$$\sqrt{\frac{2a(a+b+c)}{(a+b)(a+c)}}+\sqrt{\frac{5b(a+b+c)}{(b+c)(b+a)}}+\sqrt{\frac{10c(a+b+c)}{(c+a)(c+b)}}=6 \tag{1}$$

Với số $6$ và vai trò các biến không đối xứng thì mình nghĩ đến việc tách $6=1+2+3$. Thử bộ số $(1,2,3)$ vào và nháp xíu thì tìm được $(a,b,c)\sim (1,2,3)$ (quá đẹp rồi  :D ). Ta có 

$$VT(1)\le \frac{1}{2}\left ( \frac{2a(a+b+c)}{(a+b)(a+c)}+1+\frac{1}{2}\left ( \frac{5b(a+b+c)}{(b+c)(b+a)}+4 \right )+\frac{1}{3}\left ( \frac{10c(a+b+c)}{(c+a)(c+b)}+9 \right ) \right )$$

Đến đây ta sẽ chứng minh 

$$\frac{1}{2}\left ( \frac{2a(a+b+c)}{(a+b)(a+c)}+1+\frac{1}{2}\left ( \frac{5b(a+b+c)}{(b+c)(b+a)}+4 \right )+\frac{1}{3}\left ( \frac{10c(a+b+c)}{(c+a)(c+b)}+9 \right ) \right )\le VP(1)=6$$

Quy đồng thì thu được bất đẳng thức trên tương đương với

$$(9ab+bc+4ca)(a+b+c)\ge 36abc$$

Bất đẳng thức này luôn đúng vì theo bất đẳng thức Cauchy thì

$$(9ab+bc+4ca)(a+b+c)\ge 6\sqrt[6]{(3ab)^3\cdot bc\cdot (2ca)^2}\cdot 6\sqrt[6]{a\cdot (b/2)^2\cdot (c/3)^3}=36abc$$

Dấu bằng xảy ra khi và chỉ khi $(a,b,c)\sim (1,2,3)$, dẫn tới $(x,y,z)=\left ( 1,2,\frac{2}{5} \right )$.




#731721 Tìm $a,n$ nguyên dương để $a^{n^2+2n-1}-99$ là...

Đã gửi bởi nhungvienkimcuong on 22-11-2021 - 09:10 trong Số học

Tìm $a,n$ nguyên dương để $a^{n^2+2n-1}-99$ là số chính phương.

Giả sử $a^{n^2+2n-1}-99=b^2\ (1)$

$\bullet$ TH1: $n$ lẻ

Khi đó $n^2+2n-1=2m$. Ta viết lại $(1)$ dưới dạng $(a^m-b)(a^m+b)=99$. Tới đây tìm được

$$(a,n)\in \{(10,1),(18,1),(50,1)\}$$

$\bullet$ TH1: $n$ chẵn

Ta sẽ chứng minh không tồn tại nghiệm trong trường hợp này. Nhận thấy $n^2+2n-1=4k+3$

  • Nếu $a$ chẵn thì $b^2\equiv 5\pmod{8}$ (vô lí)
  • Nếu $a\equiv 1\pmod{4}$ thì $b^2\equiv 2\pmod{4}$ (vô lí)
  • Nếu $a\equiv 3\pmod{4}$, ta viết lại $(1)$ dưới dạng

$$a^{4k+3}+1=b^2+10^2$$

Vì $(a^{4k+3}+1)/(a+1)\equiv 3\pmod{4}$ nên tồn tại số nguyên tố $p\equiv 3\pmod{4}$ là ước của $a^{4k+3}+1$. Suy ra $p\mid b^2+10^2$, điều này dẫn tới $p\mid 10$ (vô lí)




#731580 $n\sqrt{d}\left \{ n\sqrt{d...

Đã gửi bởi nhungvienkimcuong on 11-11-2021 - 10:41 trong Số học

$\boxed{\text{Problem 2}}$ (Balkan MO 2015)

Chứng minh rằng trong $20$ số nguyên dương liên tiếp có một số nguyên dương $d$ sao cho với mọi số nguyên dương $n$, bất đẳng thức sau đúng:

$n\sqrt{d}\left \{ n\sqrt{d} \right \}>\frac{5}{2}$

Ý tưởng câu này tương tự ở đây

 

Đặt $m=\left \lfloor n\sqrt{d} \right \rfloor$. Khi đó

$$n^2d-m^2=\left \{ n\sqrt{d} \right \}(n\sqrt{d}+m)\le 2\left \{ n\sqrt{d} \right \}n\sqrt{d}$$

Ta sẽ chọn $d$ không phải là số chính phương $(1)$, dẫn tới

\[n\sqrt{d}\left \{ n\sqrt{d} \right \}>\frac{n^2d-m^2}{2}\]

Tiếp đến ta sẽ chọn $d$ phù hợp sao cho $n^2d-m^2\notin \{1,2,3,4\}$ $(2)$

 

Với $20$ số nguyên dương liên tiếp thì sẽ tồn tại $d$ có dạng $20k+15$, khi đó $d$ thỏa $(1)$ và $(2)$. Thật vậy

  • $d\equiv 3\pmod{4}$ nên $d$ không phải số chính phương
  • Tồn tại số nguyên tố $p\equiv 3\pmod{4}$ sao cho $p\mid 4k+3$, khi đó $p\mid n^2d$. Mặt khác $p\nmid a^2+1^2, a^2+2^2$ với mọi số nguyên $a$ nên $n^2d\neq m^2+1,m^2+4$
  • $5\mid n^2d$ nhưng $5\nmid a^2+2,a^2+3$ với mọi số nguyên $a$ nên $n^2d\neq m^2+2,m^2+3$

Với cách chọn $d$ như trên ta có được $n^2d-m\ge 5$ nên có được điều cần chứng minh.




#731456 $$\exists_{n}^{\infty}\;n^{...

Đã gửi bởi nhungvienkimcuong on 03-11-2021 - 14:03 trong Số học

Bạn cũng có thể thử bài này $\exists_{n}^{\infty}\;n^{2}+ 1\nmid n!\!$ HOẶC chứng minh mệnh đề của bạn với số nguyên dương $n$ thứ $k$ sao cho $n\leq 18k.$




#731306 [TOPIC] Mỗi ngày một bài toán IMO

Đã gửi bởi nhungvienkimcuong on 25-10-2021 - 15:29 trong Các dạng toán khác

Bài 8: [IMO 1987] Chứng minh rằng không tồn tại hàm $f$ nào từ tập hợp các số nguyên không âm vào chính nó thoả mãn $f(f(n))=n+1987$ với mọi $n$.

 

Bài này ta có thể tổng quát lên như sau: Tồn tại hàm số $f\colon \mathbb{N}\to \mathbb{N}$ sao cho $\underbrace{f(f(\cdots f}_{a\ \text{lần}}(n)\cdots))=n+b$ với mọi $n\in \mathbb{N}$ khi và chỉ khi $a\mid b$

 

Ở đây mình chỉ làm theo đề gốc, bài toán tổng quát các bạn dành cho các bạn  :D

Giả sử tồn tại hàm $f$ thỏa đề. Từ giả thiết dễ thấy $f(n+1987)=f(n)+1987\ \color{Red}{(1)}$.

Tiếp đến ta làm việc trong $ \mathbb{Z}_{1987}$, có được

$$f(f(x))=x,\quad \forall x\in \mathbb{Z}_{1987}$$

Xây dựng đồ thị có hướng $[V,E]$ trong đó tập đỉnh chính là các phần tử của tập hợp $\mathbb{Z}_{1987}$, có cạnh $[v_1,v_2]$ nếu $v_2=f(v_1)$. Với cách xây dựng này thì ta thấy mỗi đỉnh của đồ thị thuộc $1$-chu trình hoặc $2$-chu trình. Vì $1987$ là số lẻ nên tồn tại điểm bất điểm bất động $m\in \mathbb{Z}_{1987}$, nghĩa là $f(m)=m$.

Quay trở lại làm việc trong $\mathbb{Z}$. ta có được $f(m)=m+1987k$ với $k$ là số nguyên nào đó. Do vậy

\[f(f(m))=f(m+1987k)\overset{\color{Red}{(1)}}{=}f(m)+1987k=m+1987\cdot 2k\]

Mặt khác theo giả thiết thì $f(f(m))=m+1987$, suy ra $2k=1$ (vô lí).




#729193 CMR: $a_n\equiv 2,3,6\pmod{8}$

Đã gửi bởi nhungvienkimcuong on 28-07-2021 - 10:53 trong Số học

Tham khảo ở đây. Kết quả bài này  là $a_n\equiv 2,3\pmod{8}$.

Một bài tương tự là

$$\left \lfloor \left ( \sqrt[3]{65}-4 \right )^{-n} \right \rfloor\equiv 2,3\pmod{15}$$